Question

Given the real signal: x [n] = [1, -2,0,1], indicate the correct alternative: a) The DFT...

Given the real signal: x [n] = [1, -2,0,1], indicate the correct alternative:
a) The DFT of the signal with 6 points is [0, -1 + 1.7321j, 3 + 1.7321j, 2.3-1.7321j, -1-1.7321j]
b) The DFT of the signal with 5 points is:
[0,0,0, j, -j]
c) The DFT of the signal with 7 points does not allow the original signal to be recovered.
d) The DFT of the signal with 4 points is: x [n] = [-2, -1 + 1j, -4, -1-1j]

Homework Answers

Answer #1

Option (C) is incorrect because any DFT with N>=4 can be used to recover x[n] from it's DFT X[K].

Option (D) is incorrect because X[0]=1-2+0+1=0,but given it is -2.

Option (B) is incorrect because as x[n] is real signal , X[K] =X*[N-K]=X*[5-K],but X[1]=0 and X*[5-1]=X*[4]=(-j)*=j ,which is not equal to X[1]=0.

Option (A) is incorrect, because it has only 5 points,it can't be the 6 point DFT of x[n].

Option (A) can be modified by including the X[3]=2,it can be made correct.

Know the answer?
Your Answer:

Post as a guest

Your Name:

What's your source?

Earn Coins

Coins can be redeemed for fabulous gifts.

Not the answer you're looking for?
Ask your own homework help question
Similar Questions
A 512 point DFT X[k] of a real valued sequence x[n] has the following DFT values:...
A 512 point DFT X[k] of a real valued sequence x[n] has the following DFT values: X[0]=20+jα ; X[5] = 20+j30; X[k1]= -10 +j15; X[152] =17+j23; X[k2] = 20-j30; X[k3]=17-j23; X[480] =-10–j15; X[256]=30 + jβ; and all other values are zero. a. Determine the values of α and β. b. Determine the values of k1, k2 and k3. c. Determine the energy of the signal x[n].
DFT Question: 4. If fs = 250 and x = 2δ [n −1]−3δ [n −3]+ 5δ...
DFT Question: 4. If fs = 250 and x = 2δ [n −1]−3δ [n −3]+ 5δ [n − 4]+ 5δ [n −5], a. what does frequency of each k (which is frequency index)? b. How many total numbers of bins (each k is called bin) do you need to have if you want to represent the frequency of 15 Hz for each bin? c. If you append 10 zeros in the back of this x signal, what frequency does each...
4.2 Given a sequence x(n) for 0 ≤ n ≤ 3, where x(0) = 4, x(1)...
4.2 Given a sequence x(n) for 0 ≤ n ≤ 3, where x(0) = 4, x(1) = 3, x(2) = 2, and x(3) = 1, evaluate its DFT X(k). 4.5 Given the DFT sequence X(k) for 0 ≤ k ≤ 3 obtained in Problem 4.2, evaluate its inverse DFT x(n).
For an LTI system h[n], the output is given by y[n] = 2δ[n-1], given that x[n]...
For an LTI system h[n], the output is given by y[n] = 2δ[n-1], given that x[n] = δ[n]-2δ[n-1]+ 2δ[n-2]. a) Find the transfer function H(z) (7 Points). b) Find the difference equation of the overall system (8 Points). c) Given that the system is causal find h[n] (10 Points). d) Given that the system does not have Fourier Transform, find h[n] (10 Points).
A table of values is given for a function f(x, y) defined on R = [1,...
A table of values is given for a function f(x, y) defined on R = [1, 3] × [0, 4]. 0 1 2 3 4 1.0 2 0 -3 -6 -5 1.5 3 1 -4 -5 -6 2.0 4 3 0 -5 -8 2.5 5 4 3 -1 -4 3.0 7 8 6 3 0 Estimate f(x, y) dA R using the Midpoint Rule with m = n = 2 and estimate the double integral with m = n =...
We are given an array A of size n containing n positive and negative integers (the...
We are given an array A of size n containing n positive and negative integers (the array is indexed starting from 0). Our goal is to find two indices i and j such that 0 ≤ i ≤ j ≤ n and Pk=j k=i A[k] is maximized. Input sequence: [2, -4, 1, 9, -6, 7, -3] Output: [1, 9, -6, 7] or i = 2 and j = 5 Input sequence: [1, 2, 3, 4, 5, 6, -3] Output: [1,...
EVALUATE CNXPXQN-X FOR THE VALUES OF N,X,AD P GIVEN BELOW N=8,X-1,P=1/2 N=7,X=2,P=0.8 N=6, X=4,P=2/5
EVALUATE CNXPXQN-X FOR THE VALUES OF N,X,AD P GIVEN BELOW N=8,X-1,P=1/2 N=7,X=2,P=0.8 N=6, X=4,P=2/5
The Taylor series for the function arcsin(x)arcsin⁡(x) about x=0x=0 is equal to ∑n=0∞(2n)!4n(n!)2(2n+1)x2n+1.∑n=0∞(2n)!4n(n!)2(2n+1)x2n+1. For this question,...
The Taylor series for the function arcsin(x)arcsin⁡(x) about x=0x=0 is equal to ∑n=0∞(2n)!4n(n!)2(2n+1)x2n+1.∑n=0∞(2n)!4n(n!)2(2n+1)x2n+1. For this question, recall that 0!=10!=1. a) (6 points) What is the radius of convergence of this Taylor series? Write your final answer in a box. b) (4 points) Let TT be a constant that is within the radius of convergence you found. Write a series expansion for the following integral, using the Taylor series that is given. ∫T0arcsin(x)dx∫0Tarcsin⁡(x)dx Write your final answer in a box. c)...
Given signal x(t) = sinc(t): 1. Find out the Fourier transform of x(t), find X(f), sketch...
Given signal x(t) = sinc(t): 1. Find out the Fourier transform of x(t), find X(f), sketch them. 2. Find out the Nyquist sampling frequency of x(t). 3. Given sampling rate fs, write down the expression of the Fourier transform of xs(t), Xs(f) in terms of X(f). 4. Let sampling frequency fs = 1Hz. Sketch the sampled signal xs(t) = x(kTs) and the Fourier transform of xs(t), Xs(f). 5. Let sampling frequency fs = 2Hz. Repeat 4. 6. Let sampling frequency...
Given that 1/ 1 − x = ∑ n = 0 x^n with convergence in (−1,...
Given that 1/ 1 − x = ∑ n = 0 x^n with convergence in (−1, 1), find the power series for 1 x with center 5. ∞ ∑ n = 0 Identify its interval of convergence. The series is convergent from x = , left end included (enter Y or N): to x = , right end included (enter Y or N):